Números complejos

Problemas resueltos

Problema 1

Dada una constante real positiva, $a$, y el conjunto

$$ M_a = \left\{z\in\mathbb{C}^* : \left|z+\dfrac{1}{z}\right|=a \right\}, $$

donde $\mathbb{C}^* = \mathbb{C}\backslash{(0,0)}$, encuentre los valores mínimo y máximo de $|z|$ cuando $z\in M_a$.

Solución: hemos de ser capaces de extraer información sobre el módulo de $z$, $|z|$, a partir de la expresión de la ecuación que define al conjunto $M_a$. Por las propiedades de la conjugación, sabemos que $z\overline{z} = |z|^2$, de manera que una posible estrategia a seguir sería elevar al cuadrado ambos miembros de la ecuación que define a $M_a$ y desarrollar la expresión resultante mediante las propiedades de la conjugación. Así,

$$ \begin{aligned} a^2 &= \left|z+\dfrac{1}{z}\right|^2 = \left(z+\dfrac{1}{z}\right)\overline{\left(z+\dfrac{1}{z}\right)} = \left(z+\dfrac{1}{z}\right)\left(\overline{z}+\dfrac{1}{\overline{z}}\right)\\ &= z\overline{z} + \dfrac{z}{\overline{z}} + \dfrac{\overline{z}}{z} + \dfrac{1}{z\overline{z}} = z\overline{z} + \dfrac{z^2 + \overline{z}^2}{\overline{z}z} + \dfrac{1}{z\overline{z}}, \end{aligned} $$

pero recordemos que $z\overline{z} = |z|^2$, por lo que la anterior ecuación quedaría

$$ a^2 = |z|^2 + \dfrac{z^2+\overline{z}^2}{|z|^2}+\dfrac{1}{|z|^2}, $$

y multiplicando ambos miembros por $|z|^2$, número que sabemos es distinto de cero porque $z\in\mathbb{C}^*$, tenemos

$$ a^2|z|^2 = |z|^4 + z^2 + \overline{z}^2 + 1. $$

Ahora bien, a primera vista, la anterior expresión parece que quiere conducirnos hacia alguna especie de ecuación bicuadrada en $|z|$. Efectivamente, si reordenamos los términos de manera adecuada, llegamos a

$$ |z|^4 - a^2|z|^2 + 1 = - (z^2 + \overline{z}^2) $$

y si completamos el cuadrado del miembro derecho,

$$ \begin{aligned} |z|^4 - a^2|z|^2 + 1 &= - (z^2 + \overline{z}^2 + 2z\overline{z} - 2z\overline{z})\\ &= - (z + \overline{z})^2 + 2z\overline{z}\\ &= - (z + \overline{z})^2 + 2|z|^2, \end{aligned} $$

es decir,

$$ |z|^4 - (a^2+2)|z|^2 + 1 = - (z+\overline{z})^2 \leq 0. $$

Por lo tanto, tras realizar operaciones algebraicas sobre la ecuación que define al conjunto $M_a$, hemos llegado a que la expresión de cierta ecuación bicuadrada en $|z|$ debe ser menor o igual que cero. Investiguemos si de ella podemos extraer alguna condición sobre el módulo de $z$ que nos permita dar respuesta al problema planteado.

Resolviendo $|z|^4 - (a^2 + 2)|z|^2 + 1 = 0$ y teniendo en cuenta que es positivo el coeficiente asociado a $|z|^4$, tenemos que $|z|^4 - (a^2 + 2)|z|^2 + 1 \leq 0$ si, y solo si,

$$ |z|^2\in \left[ \dfrac{2+a^2 - \sqrt{a^4+4a^2}}{2}, \dfrac{2+a^2 + \sqrt{a^4+4a^2}}{2} \right], $$

que es equivalente a decir que

$$ |z|\in \left[ \dfrac{-a + \sqrt{a^2+4}}{2}, \dfrac{a + \sqrt{a^2+4}}{2} \right]. $$

Veamos esta última equivalencia en detalle, pues puede no resultarnos trivial a primera vista. La idea aquí es representar ambos extremos del intervalo que hemos obtenido para $|z|^2$ como cuadrados de ciertas expresiones. Si somos capaces de llevar a cabo tal tarea, únicamente bastará aplicar la raíz cuadrada (que, recordemos, es una transformación monótona creciente) para, de esta manera, llegar a la conclusión obtenida para $|z|$.

Centrémonos en la expresión del extremo inferior del intervalo definido para $|z|^2$ (el razonamiento a seguir sería análogo para el superior). Por un lado,

$$ 2+a^2 - \sqrt{a^4 + 4a^2} = 2+a^2 - a\sqrt{a^2 + 4}, $$

expresión que nos invita a experimentar con el desarrollo de $(-a + \sqrt{a^2 + 4})^2$. Así,

$$ \begin{aligned} (-a + \sqrt{a^2+4})^2 &= a^2 + a^2 + 4 - 2a\sqrt{a^2+4}\\ &= 2a^2+4 - 2a\sqrt{a^2+4}\\ &= 2(a^2+2-a\sqrt{a^2+4})\\ &= 2(a^2+2-\sqrt{a^4+4a^2}), \end{aligned} $$

de forma que

$$ \dfrac{2+a^2 - \sqrt{a^4+4a^2}}{2} = \dfrac{(-a + \sqrt{a^2+4})^2}{2\cdot 2} = \left(\dfrac{-a + \sqrt{a^2+4}}{2}\right)^2. $$

Por tanto, hemos llegado a que

$$ \begin{aligned} \min{|z|} &= \dfrac{-a+\sqrt{a^2+4}}{2},\\ \max{|z|} &= \dfrac{a+\sqrt{a^2+4}}{2}, \end{aligned} $$

y los valores extremos se alcanzarán cuando se dé la igualdad en la inecuación considerada, es decir, cuando $-(z+\overline{z})^2 = 0$, que equivale a la condición $z=-\overline{z}$. Así pues, los valores extremos se alcanzarán para los números complejos $z\in M_a$ tales que $z=-\overline{z}$.

Problemas propuestos

Problema 2: halla el conjunto de puntos de la variable compleja $z$ tal que $RE(z^2) > 2$.

Problema 3: halla la curva definida por

$$ RE\left(\frac{1}{z}\right) = \frac{1}{4}. $$

Problema 4: calcula $\sqrt{-15-8i}$.

Problema 5: resuelve $z^2 + (2i - 3)z + 5-i = 0$.

Problema 6: halla las raíces quintas de la unidad.

Problema 7: sean $r_i$, con $i=1,\ldots,5$, las raíces quintas de la unidad. Halla el valor de

$$ A = r_1^n + r_2^n + r_3^n + r_4^n + r_5^n. $$

Problema 8: calcula $(1+\sqrt{3}i)^n + (1-\sqrt{3}i)^n$.

Problema 9: sea

$$ z = \left(\frac{-1+\sqrt{3}i}{2}\right)^n + \left(\frac{-1-\sqrt{3}i}{2}\right)^n. $$

Prueba que $z=2$ si $n$ es múltiplo de $3$ y $z=-1$ en cualquier otro caso.

Problema 10: escribe en forma binómica $e^{\sqrt{i}}$.

Problema 11: si

$$ z + \frac{1}{z} = 2\cos{t}, $$

calcula

$$ z^n + \frac{1}{z^n}. $$

Problema 12: halla un número complejo que cumpla $z^5 = \overline{z}$.

Problema 13: calcula $3^i$.

Problema 14: calcula $\log_{1 + i}{(8 - 8i)}$.

Problema 15: resuelve

  • (a) $\cos{(z)} = 2$.
  • (b) $\sin{(z)} = 4$.

Problema 16: resuelve $\cosh^2{(z)} - 3\sinh{(z)} + 1 = 0$.

Problema 17: dados los puntos $A(1, 2)$ y $B(3,3)$, determina, como número complejo en forma binómica, los vértices de un triángulo equilátero con centro $A$, sabiendo que $B$ es uno de sus vértices.

Problema 18: determine los vértices de un cuadrado sabiendo que

  • (a) Su centro es el punto $(2, 3)$.
  • (b) Si se traslada el centro al origen, se gira un ángulo de $60$ grados, en sentido positivo, y se reducen los lados del cuadrado a la mitad; los vértices del nuevo cuadrado son los afijos de las raíces de un polinomio de grado cuatro, con coeficientes reales, que tiene la raíz $x = 1$.

Problema 19: los afijos $z_1, z_2, z_3, z_4, z_5$ y $z_6$ son los vértices consecutivos de un hexágono regular. Sabiendo que $z_1 = 0$ y que $z_4 = 4 + 6i$, halla $z_2, z_3, z_5$ y $z_6$.

Problema 20: dados tres complejos $z_1, z_2$ y $z_3$, demuestra que si forman un triángulo equilátero, se cumple que

$$ z_1^2 + z_2^2 + z_3^2 = z_1z_2 + z_1z_3 + z_2z_3. $$

Problema 21: siendo $a$ un número complejo fijo, determina (en función de $a$) los posibles números complejos $z$, tales que las imágenes en el plano complejo de los afijos $a^2z, az^2$ y $z^3$ son vértices de un triángulo equilátero.

Problema 22: calcula

  • (a) $\prod_{k=1}^{n-1}{\left(e^{\frac{2k\pi i}{n}} - 1\right)}$.
  • (b) $\prod_{k=1}^{n-1}{\sin{\left(\frac{k\pi}{n}\right)}}$.

Problema 23: dado un número real $a$ y un número natural $n$, calcula

$$ \cos{(a)} + \cos{(2a)} + \cos{(3a)} + \cdots + \cos{(na)}. $$

Problema 24: calcula

$$ \sum_{n=0}^{\infty}{\frac{\cos{(n\alpha)}}{2^n}}. $$